Sports Medicine Daily Q

¡Supera tus tareas y exámenes ahora con Quizwiz!

A 20-year-old female who is joining an intramural team at her college was noted to have an unusual murmur on her sports physical before the season began. The doctor thought he heard a continuous murmur at the left upper sternal border associated with a slightly widened pulse pressure and brisk to bounding pulses. Which of the following is the most likely diagnosis? A. VSD B. ASD C. Coarctation of the aorta D. Patent ductus arteriosus (PDA)

(D) Patent ductus arteriosus (PDA). Explanation: Many adult patients with patent ductus are asymptomatic, depending on the size of the left-to-right shunt and the size of the ductus. Frequently, the condition is discovered by the unusual quality of a continuous murmur at the left upper sternal border that can sound like an innocent venous hum. Because a patent ductus is an aortopulmonary runoff, however, the pulse pressure frequently is widened, and the pulses are brisk to bounding. Today, most lesions of ductus can be closed in the catheterization laboratory without surgery.

Internal impingement commonly occurs in overhead athletes and is very common amongst elite baseball pitchers. In which phase of throwing does this pathologic process occur? 1. Wind-up 2. Early cocking 3. Late cocking 4. Deceleration 5. Follow-through

Answer: Internal impingement refers to the impingement within the glenohumeral joint which occurs as the posterosuperior glenoid labrum makes contact with the greater tuberosity, causing impingement on the posterior rotator cuff. This occurs commonly among baseball pitchers during late cocking and early acceleration as the shoulder joint reaches it's maximum external rotation.Drakos et al studied internal impingement in overhead athletes. In their review, they describe how adaptations to throwing including increased external rotation, increased humeral and glenoid retroversion, and anterior laxity, all predispose these individuals to internal impingement. They recommend initial nonoperative treatment, with a focus on increasing the range of motion and improving scapular function. If this fails, surgical treatment should address microinstability to ensure good outcomes.

Your star quarterback gets sacked from behind, lands on a flexed knee with plantarflexed foot. He comes off the field limping, reporting a popping sensation at the time of the injury. What is the most likely injury based upon his history? A. ACL tear B. PCL tear C. MCL tear D. LCL tear E. Meniscal tear

B, PCL tear.

While completing the PPE paperwork on a Down syndrome athlete, you review the report of the lateral c-spine x-rays to screen for atlanto-axial instability. You recall that a normal atlantodens interval (ADI) is A. 0 mm B. <2.5 mm C. >4.5 mm D. >6.0 mm

B. AAI is screened with lateral c-spine films in flexion, extension, and neutral. The ADI, the distance between the odontoid process of the axis and the anterior arch of the atlas, is normally less than 2.5 mm. Greater than 4.5 mm is abnormal. If greater than 6.0 mm, the athlete should be restricted from all strenuous activities and evaluated for surgical intervention.

You are seeing a new patient for a pre-participation exam and this athlete would like to be cleared to scuba dive. Which statement is true regarding scuba diving participation? A. An athlete with a well-controlled seizure disorder who is on a stable dose of medication and has been seizure-free for 6 months is safe to be cleared B. An athlete with a previous spontaneous pneumothorax can be cleared for shallow dives only C. An athlete with sickle cell trait has no more risk of hypoxia than the average diver and is safe to be cleared D. An athlete with myringotomy tubes in place for 3 months is safe to be cleared E. An athlete with an active otitis media if treated with antibiotics is safe to be cleared

Correct: C Anti-epileptic medication is an absolute contraindication to scuba even with ideal seizure control since seizure activity underwater is lethal. A previous spontaneous pneumothorax is an absolute contraindication to scuba and it is important to remember that the greatest change in water pressure occurs in the first 10 feet. The risk of pneumothorax is actually highest at shallow depths. Alveolar rupture while breathing compressed air can occur in depths as shallow as 4 feet. If an athlete with sickle cell trait were to lose his/her source of oxygen while diving (out of air, drowning, equipment failure, etc.), then the cells would sickle. However, the low-oxygen tensions necessary to produce this phenomenon would have caused brain damage long before the red cells were involved. The person with sickle cell trait then is not at any greater risk from hypoxia (oxygen deficiency in body tissues) than the ordinary diver and is qualified for diving (if trained and physically fit otherwise). Myringotomy tubes allow water to enter the middle ear and this can induce cold water calorics causing potentially lethal nausea, vomiting and vertigo. Otitis media prevents normal eustachian tube function which in turn prevents normal middle ear ventilation. Normal eustachian tube function allows the equalization of air pressure between the middle ear and the external water pressure. Without this ability, the tympanic membrane is at increased risk of rupture.

A 14 year old boy nears the end of a 5 minute mile track race during an indoor track meet. He has sudden sharp pain over the left hip just proximal to the inguinal ligament. He has mild nausea and even one episode of emesis. In the office his exam shows left low back tenderness and limited forward flexion at the waist without neurologic or radicular findings. He also has tenderness to direct and firm palpation over the left superior ilium along its anterior third. His physical exam also reveals no hernia, and the abdomen is benign. His plain radiographs and an abdominal CT in the emergency room are normal. MRI reveals a mild avulsion of the apophysis over the left superior ileum as well as some mild edema indicating injury to the left quadratus lumborum. The plain film was read again showing the bony avulsion found on his MRI. Proper recommendations include: A. No running until radiologic healing is proven by plain x-ray B. Refer to orthopedic surgeon C. Conservative care, relative rest, return to running when pain has resolved and a full range of motion has returned D. Limit his passive hip flexion to allow healing E. Advise no further sprinting or racing on the track during this T

Correct: C There are five apophyses near the hip. These secondary growth centers fuse by about age 25. Forceful or sudden traction at these sites can cause an apophysitis with pain and limitation of motion. Injury can also cause an avulsion of the apophysis itself. Treatment is conservative. There is no need to limit motion in general. Specific rehabilitation protocols have been described, and athletes have done well with them. Athletes may return to full activity with normal pain-free range of motion and normal strength. This teen had imaging evidence of two separate conditions on his MRI: an avulsion of the apophysis and an acute muscle strain. These conditions may be handled by primary care physicians. Surgery is generally not required. An MRI is usually not necessary to make the diagnosis but may be helpful to diagnose or rule out other pathology. Running the mile, bony injuries, and GI pathology can all cause nausea and vomiting.

What is the most common virus to cause myocarditis and sudden cardiac death?

Coxsackie B virus

What are the Ottawa rules for determining x-ray imaging of the acute knee? What are the Pittsburgh Decision rules for determining x-rays of the knee, and is there a significant difference in sensitivity or specificity compared to the Ottawa rules?

Ottawa Knee rules for x-ray imaging: > 1. age 55 or over; > 2. isolated tenderness of the patella; > 3. tenderness at the head of the fibula; 4. inability to flex to 90 degrees; > 5. inability to walk four weight-bearing steps both immediately and in the emergency department. Bonus answer: Pittsburgh Decision rules > 1. Blunt trauma or a fall as mechanism of injury plus either of the following: > 2. Age younger than 12 years or older than 50 years > 3. Inability to walk four weight-bearing steps in the emergency department >> Efficacy of the Ottawa Knee Rules: > Sensitivity: 97% > Specificity: 27% > Reduced the use of knee radiographs by 28% >> Efficacy of the Pittsburgh Decision Rules: > Sensitivity: 99% (1) > Specificity: 60% (1) > Reduced the use of knee radiographs by 52%

What is the source of energy the body uses for immediate short burst activity during the first 5-6 seconds of activity?

The ATP/phosphocreatine system.

what is the difference between type 1 and type 2 Decompression sickness?

Type 1 DCS shows pain in the joints, marbling of skin and lymph nodes Type 2 DCS will also show neurologic, inner ear and cardiopulmonary symptoms

A 25-year-old track runner notices an audible popping sensation and pain across her lateral knee and thigh when she is running, especially downhill. She has increased the duration and intensity of her workouts to prepare for an upcoming marathon. On physical exam, there is a snapping sensation at the lateral hip with rotation of the hip. She also experiences tenderness on palpation of the trochanteric bursa. Which of the following is the most likely diagnosis? A. Anterior cruciate ligament tear B. Iliotibial band syndrome C. Lateral collateral ligament injury D. Patellofemoral pain syndrome E. Trochanteric bursitis

(B) Iliotibial band syndrome. Explanation: Iliotibial band syndrome is the most common cause of lateral knee pain among athletes. It develops as a result of inflammation of the bursa surrounding the illiotibial band and usually affects athletes involved in sports that require continuous running or repetitive knee flexion and extension. This condition is most common in long-distance runners and cyclists. The illiotibial band is the condensation of fascia formed by the tensor fascia lata and the gluteus medius and minimus muscles. It is a wide, flat structure that originates at the iliac crest and inserts on the lateral aspect of the proximal tibia. Typically, the patient presents with an insidious onset of lateral knee pain that is present during running. Pain is localized over the lateral epicondyle, and the athlete is able to localize the lateral knee pain to approximately 2cm above the lateral joint line. Pain is experienced usually when the athlete climbs stairs or runs downhill. Early in the course of the injury, the pain usually resolves after running. Conservative measures such as stretching, foam roller exercises, rest, ice, and NSAIDS are the mainstay of treatment. If symptoms to do not improve, an MRI may be warranted to rule out other conditions such as a ligament tear.

You are seeing a 6 year old for a limp. It was initially intermittent but has been becoming more consistent. He does not complain of pain. There was no known injury and he does not play organized sports. He has no fever, no preceding illness, and is otherwise well. The most likely diagnosis is: A. Slipped capital femoral epiphysis B. Legg Calve Perthes disease C. Neoplasm D. Juvenile Idiopathic Arthritis E. Toxic synovitis

(B) Legg Calve Perthes disease. Legg Calve Perthes disease is an avascular necrosis of the femoral head which occurs in children ages 2-12 (more commonly ages 4-8). It is more common in boys and classically presents as a painless limp. Hip radiographs (AP and frog leg lateral) are helpful in making the diagnosis. Treatment includes limiting weight bearing, bracing, and surgery. (A) Slipped capital femoral epiphysis classically presents as hip or knee pain and limp in an overweight teenager. It is diagnosed by getting AP and frog leg lateral radiographs and the treatment is usually surgical. (C) Neoplasm: in this age, leukemia could present with a limp secondary to bone pain, although usually fever is present. Primary bone neoplasms (osteosarcoma and Ewing's sarcoma) usually occur in older children/ teenagers. (D) Juvenile Idiopathic Arthritis can present with a limp. Often the symptoms are worse in the morning and improve with activity. When the hip is involved, it is often held in a position of flexion, abduction, and external rotation and there is often pain on range of motion. The child is also more likely to have signs of systemic illness. (E) Toxic synovitis usually occurs in a younger child and presents as hip or knee pain and a limp. There is often a history of a preceding viral illness. The child will resist rotation of the hip. Treatment is supportive and the condition self resolves.

Which of the following is true of weight loss and metabolism during exercise? A. Maximal fat oxidation occurs at high intensity exercise. B At maximal exercise intensity, blood flow increases to adipose tissues. C. Excess post exercise oxygen consumption is highest after long duration of low intensity exercise D. Training adaptations lead to decreased use of fat at the same exercise intensity. E. Fat utilization is decreased during high intensity exercise.

(E) Fat utilization is decreased during high intensity exercise. Explanation: There is a counterintuitive drop in utilization of fat during high intensity exercise that is caused by several factors, including the shunting of blood flow away from adipose tissue during maximal exercise intensity. This causes trapping of fatty acids in the adipose capillary beds, preventing fatty acids from being carried to muscle to be used. (A) Fat oxidation is maximal at low to moderate levels of exercise intensity. (B) Blood is shunted from adipose tissues during exercise at maximum intensity. (C) Excess post exercise oxygen consumption refers to an elevated metabolic rate after exercise that results from energy utilized for muscle cell recovery and glycogen replacement. It is greatest when exercise intensity is high and greater after high intensity interval training compared to exercise for a longer duration at lower intensity. (D) Training adaptations lead to increased use of fat at the same exercise intensity.

Your patient presents to you for follow up of a FOOSH mechanism and radial sided wrist pain. He was seen in an urgent care at the time of the injury, had negative x-rays of the wrist, and diagnosed with a sprain. He is now 10 days out from the injury and is no better. Your exam suggests a scaphoid fracture. Proper methods of diagnosis and treatment include all of the following EXCEPT: repeat plain films, including scaphoid view, of the wrist CT scan of the wrist if the plain films are normal/negative for a nondisplaced waist fracture can heal well, but will require 8-12 weeks of immobilization short arm thumb spica casting for distal or waist fractures operative fixation for proximal pole or displaced fractures of any zone

2 - All of the above are true except for option 2. A repeat plain film at 10-14 days post injury may now demonstrate a fracture that was not visible initially. Although controversial to an extent, immobilizing the thumb to minimize articular movement at the scaphoid makes clinical sense over leaving the thumb out of the cast. It is not necessarily widely accepted any longer to immobilize in a long arm cast. Of the non displaced fractures, distal pole injuries can take about 6 weeks to resolve, waist injuries anywhere from 8-12 weeks, and proximal pole injuries from 12-24 weeks, with cast immobilization. Obviously, displaced fractures require anatomic reduction, and proximal pole fractures are inherently risky for AVN or nonunion due to lack of adequate vascular support. MRI is favored over CT imaging, as it is non ionizing, can identify occult fractures via marrow edema signal changes. MRI can approach 100% sensitivity and specificity for occult scaphoid fractures.

In response to intense exercise, catecholamine release will occur. These hormones can lead to several effects in the athlete. Which of the following is due to alpha receptor effect? A. Vasoconstriction B. Cardiac acceleration C. Lipolysis D. Bronchodilatation E. Increased myocardial contractility

A Cardiac acceleration, increased myocardial contractility, and lipolysis are beta1 receptor effects (B,C,E). Bronchodilatation is a beta2 effect (D).

What is the role of calcium in muscle contraction? A. calcium binds troponin, moving tropomysin and allowing crossbridge linkages and contraction B. calcium binds troponin allowing for release of ATP and therefore initiating contraction C. calcium binds tropomysin, moving troponin and allowing crossbridge linkages and contraction D. calcium binds tropomysin allowing for release of ATP and therefore initiating contraction

A Muscle contraction is triggered by an electrical impulse involving acetylcholine release which subsequently results in calcium release from the sacoplasmic reticulum. Once released calcium binds troponin which causes tropomysin to move and therefore allowing activation of cross linkages and contraction.

A patient presents to your office unable to dorsiflex his great toe. Which of the following is true? A. The extensor hallucis longus which inserts on base of the distal phalanx of the great toe is the muscle responsible for extending the great toe B. The motor function for this is from L4 and L5 C. The muscles that allow this action are all contained in the lateral compartment of the lower leg D. The muscles that allow this action are inervated by the tibialis anterior nerve

A The extensor hallus longus originates on the medial fibula in the interosseous membrane and it inserts onto the base of the distal phalanx of the great toe. It is inervated by the deep peroneal nerve, not the tibialis anterior nerve and it is inervated by L5, not L4. The muscles that act as extensors of the foot and toe are the anterior compartment of the lower extremity. The lateral compartment contains the proneous longus, proneous brevis and the peroneal nerve.

A 26 year old African American female presents to the medical treatment tent you are staffing at a large cross country ski race in upper Wisconsin. She is complaining of painful edematous purple lesions on her face. She is in excellent health, an avid cross country runner from southern Illinois. She denies pregnancy or any medical problems. She does not seem to be in any acute distress. She and her friends have been taking "nips" out of a pocket flask containing Blackberry brandy. Which of the following is true? A. She has classic Pernio or chilblain B. She should immediately stop the race and be transported to the main medical tent 10 kilometers away via ambulance C. She can go back out after applying protective UV cold barrier ointment on her face D. She should quickly rewarm her face by sitting next to the propane gas warmer in the tent E. It is best to warm her face slowly using cool water then to slowly apply heated water to prevent further tissue damage

A The patient has classic Pernio, or chilblain, which is characterized by localized inflammatory lesions that result from acute or repetitive exposure to cold. The lesions are edematous, often purple, and are most common in young women. It is one of the milder forms of a cold injury. First degree frostbite is characterized by a central area of pallor and anesthesia of the skin surrounded by edema. A second degree frostbite is recognized by blisters containing a clear milky fluid surrounded by edema and erythema. Third degree frostbite differs from second degree frostbite as the injury is deeper and blisters are hemorrhagic. Alcohol use predisposes cold injury. Risk factors include smoking, previous cold injury and exposure of hands and arms to vibration. African-American women may be at increased risk of cold injury. This patient does not have any other physical signs that would require immediate transportation to the main medical tent. The best medical advice would be to simply stop the race at this point, and get her to a warm environment. In some areas protective ointments applied to the face have been advocated, this may actually increase the risk of a cold injury. The area should be warmed as soon as possible and it is best to get the patient into a warm environment and remove wet clothing. Stoves or open fires used to rewarm frostbitten tissue is not recommended as the tissue is insensitive and thermal injury can occur. If necessary, it is best to rewarm the area in a water bath 40°C to 42°C which feels warm, but not hot, to the patient.

45 year old tennis player presents with 6 weeks of low back pain with radiation to the left big toe made worse with bending over to tie his shoes. He wants to do physical therapy and you write for which back program to reduce his current symptoms of pain? A. McKenzie exercises B. Williams exercises C. Back school D. Lumbar traction

A This patient likely has a herniated disc with radicular symptoms to the foot. Flexion maneuvers make his symptoms worse so Williams' exercises will exacerbate his symptoms. Back schools in a recent review published in Annals of Internal Medicine showed no decrease in pain or recurrence of low back pain but improved short term recovery and return to work. The same review in addition to a Cochrane Review, found little evidence to support the use of lumbar traction to decrease pain. McKenzie exercises are hyperextension maneuvers that reduce symptoms associated with disc disease.

A Trendelenburg gait would most likely be caused by which of the following lumbar conditions? 1. L3/4 far lateral disc herniation 2. L3/4 central disc herniation with impingement on the bilateral descending nerve roots 3. L4/5 far lateral disc herniation 4. L5/S1 far lateral disc herniation 5. L5/S1 paracentral disc herniation

A Trendelenburg gait is caused by gluteus medius weakness. Gluteus medius is innervated by L5. Therefore a L5/S1 far lateral disc herniation could cause this condition. Gluteus medius originates for the dorsal ilium inferior to iliac crest and inserts to the lateral and superior surfaces of greater trochanter. It is the major abductor of thigh, and also functions to help to rotate the hip medially and laterally. Arterial supply is by the superior gluteal artery. It is mediated by the superior gluteal nerve, which is primarily innervated by L5. L5 also contributes to ankle dorsiflexion (combined with L4), great toe extension, and sensory over lateral calf and dorsal foot.

At what wet bulb globe temperature would you, as race medical director, recommend canceling a race per IMMD guidelines

A WBGT greater than 82 degrees is recommended as the threshold for canceling races

What is a white WBGT?

A WBGT less than 50 is considered a white temp and is the threshold for concerns for hypothermia in runners. Especially if wet or windy

Which of the following statements is true regarding scapular fractures? A. Reduction of an isolated displaced glenoid neck fracture is usually not necessary to achieve a good clinical outcome B. Non-surgical treatment is an option for glenoid fractures that involve 50% or less of the articular surface C. Non-operative treatment of glenoid fractures consists of immobilization with a sling and swathe for at least 2 weeks D. Non-operative treatment of glenoid fractures consists of immobilization with a sling for at least 2 weeks E. Sports-related injury is responsible for almost half of the cases of scapular fractures

A. Because of the large range-of-motion of the glenohumeral joint reduction of an isolated displaced glenoid neck fracture is usually not necessary to achieve a good clinical outcome. Association of a glenoid neck fracture with a displaced clavicle fracture or coracoclavicular ligament tear usually results in an unstable injury and therefore will require operative management. Surgical treatment should be strongly considered for involvement of more than 25% of the articular surface. Non-operative treatment of glenoid fractures involves use of a sling for comfort and early mobilization. Motor vehicle accidents are the most common cause of scapular fractures; they are relatively rare in sports since they require a high level of force to occur.

All of the following are physiologic adaptations to aerobic conditioning except: A. Increased ventricular wall thickness and decreased internal diameter B. Larger stroke volume C. Lower-end systolic volume D. Lower resting heart rate

A. Cardiac adaptations include increased ventricular wall thickness and INCREASED internal diameter, leading to increased stroke volume, reduced end diastolic volume and a reduced heart rate at rest.

During the second football practice of the day on day three of the college preseason football camp, an offensive lineman is found sitting on the ground unwilling to stand up. He states his left calf is cramping and that he feels lightheaded and exhausted. You suspect possible exertional heat stroke. Which of the following statements about this condition is true? A. Axillary, oral, or a rectal temperature greater than 104 F (40C) establishes the diagnosis of exertional heat stroke B. This condition occurs randomly without warning and can not be predicted C. Cold/ice water immersion is an effective way to treat exertional heat stroke D. There are two patterns of presentation: sodium depletion and water depletion

Answer C is correct. Cold/ice water immersion has received much critism from both medical professionals and industry that propose more superior cooling methods. Potential complications have been raised including cardiovascular shock, hypothermia due to excessive cooling, inadequate access for other medical interventions, peripheral vasoconstriction, and shivering. While important to consider these issues, none of them have been proven as valid reasons not to utilize rapid cooling via cold/ice water immersion to treat EHS. EHS is defined as a rectal temperature greater than 40-C (104-F) accompanied by symptoms or signs of organ system failure, most frequently central nervous system dysfunction. Answer A is incorrect because temperature devices that assess a site on the outside of the body should not be used for the diagnosis of EHS in an athlete who has been exercising in the heat as none have been proven to be accurate. Therefore, oral and axillary temperatures should not be used. Two viable options to establish CORE body temperature currently exist: 1. Rectal Thermometer - Unfortunately, an obvious drawback to rectal temperatures is the invasive nature and the lack of privacy to perform this technique on the athletic field. 2. The ingestible thermistor is a second viable field measure. These devices transmit a signal that is obtained by a receiver that is held near the athlete. They provide a rapid assesment of CORE temperature, but they are expensive and must be ingested before the problem arises. Answer B is incorrect because studies of exertional heat stroke have identified multiple risk factors that can allow health professionals to recognize predictable patterns allowing for primary prevention of this condition. Common risk factors include low physical fitness or physical effort unmatched to physical activity, underlying illness, improper acclimatization, heat load corresponding to green flag or above WBGT >27C), training at hottest hours, dehydration and sleep deprivation. Answer D is incorrect, this describes two patterns that can cause exertional heat exhaustion no stroke.

The triangular fibrocartilage complex is composed of: 1. extensor carpi ulnaris tendon sheath 2. articular disc 3. ulnotriquetral ligament 4. all of the above.

Answer: 4. All of the above

Which of the following statements about open and closed kinetic chain exercises is correct? A. Open kinetic chain exercises occur when the distal aspect of the extremity is fixed and cannot move B. Closed kinetic chain exercises typically involve functional weight-bearing and sport-specific activities C. Knee extensions and straight leg raises are examples of closed kinetic chain exercises D. During open kinetic chain exercises, motion occurs simultaneously at all joints comprising the kinetic chain E. Closed kinetic chain exercises produce shearing forces, while open kinetic chain exercises produce compressive forces

Answer: B Explanation: Open kinetic chain exercises involve free movement of the distal segment and are typically non-weightbearing. Examples include knee extensions and straight leg raises. Conversely, closed kinetic chain exercises involve fixation of the distal aspect of the extremity, and they are important during functional weightbearing activities (B). (A) is incorrect because the answer choice describes closed kinetic chain exercises. (C) is incorrect because knee extensions and straight leg raises are examples of open kinetic chain exercises. (D) is incorrect since simultaneous motion at all joints occurs during closed kinetic chain exercises. (E) is incorrect because open chain exercises produce shearing forces, while closed chain exercises produce compressive forces.

A day after being struck with a pitched ball on the ulnar aspect of the left wrist and hand, a professional baseball player develops "pins and needles" in the small and ulnar half of his ring fingers. He finds it extremely difficult to grab the bat to participate in batting practice. After x-rays demonstrate no acute abnormalities of the left wrist and hand, he is diagnosed with Guyon's canal syndrome. What two bones form Guyon's canal? A. Pisiform and Triquetrum B. Pisiform and Hamate C. Hamate and Lunate D. Triquetrum and Lunate

Answer: B Guyon's canal syndrome is entrapment of the ulnar nerve as it passes through a tunnel in the wrist called Guyon's canal. The canal is formed by the most lateral bones of the proximal and distal carpal rows, the pisiform and hamate respectively, and the ligament that connects them. The ulnar nerve is accompanied by the ulnar artery as it passes through this canal. Symptoms can include a sensation of pins and needles in the small and ulnar half of the ring fingers, decreased sensation in the same distribution as well as weakness of the small muscles of the palm and the muscle that pulls the thumb towards the palm.

What is periodization as it relates to endurance training?

Answer: Structuring of training hours for a given cycle to produce a progressive increase of training stress and performance

A 21 year old type 1 diabetic athlete begins training for a 50 mile bike ride with a partner. She uses an insulin pump and is experienced with running cross country in high school. During her first 30 mile ride she experiences symptoms of hypoglycemia at 25 miles and almost falls before stopping. She is confused and her BS is 40. What is the most appropriate immediate action? A. Eat a banana or sports bar B. Administration of glucagon by her partner C. Drink a carbohydrate sports drink then quickly resume riding to reach a safe destination D. Drink 8 ounces of water to improve volume status

B Glucagon has most rapid onset of action in a confused, uncooperative athlete. Hyperinsulinemia due to the pump is the cause. A reduction of infusion by 50% is needed for longer bouts of exercise. A solid carbohydrate food will be absorbed too slowly to prevent potential serious CNS complications in this emergency. Resumption of exercise without adjusting the continuous pump plus a carbohydrate bolus will result in progressive hypoglycemia and CNS decline. Dehydration may exist for various reasons but carbohydrate fuel is absolute necessity in this situation.

Which statement below is true regarding preadolescents and well structured weight lifting programs? A. strength training increases both muscle strength and hypertrophy in preadolescents B. strength training increases muscle strength, but not hypertrophy in preadolescents C. strength training is considered harmful to maturation, but beneficial to growth in preadolescents D. strength training can have a negative impact on maturation and growth in preadolescents

B Preadolescent resistance training programs that include protocols with weights and resistance machines, and have low instructor to participant ratio can have significant improvement in strength without hypertrophy of muscle or deliterious effects on growth or maturation.

An 18 year old returns from a trip to Colorado with ankle pain and an antalgic gait. She was treated initially for a severe ankle sprain after eversion injury during snowboard lessons. Ankle radiographs reveal a lateral process fracture of the talus that is typical in snowboarding injuries, and a CT scan verifies the non-displaced position of the small fragment. Appropriate treatment consists of A. Ankle rehab and return to activity if the CT scan shows a Hawkin's sign B. Ankle splint, weight bearing as tolerated, and aggressive ankle rehabilitation C. Nonweightbearing in a cast for 4-6 weeks followed by progressive weight bearing and ankle rehabilitation D. Walking cast for 4-6 weeks, then ankle rehabilitation E. Emergent orthopedic consultation because of tenuous blood supply to the talus

C A small non-displaced fracture of the lateral process may be appropriately treated with cast immobilization for 4-6 weeks. Since the lateral process supports 16-17% of the body's weight through the leg, early weight bearing risks displacement and surgical fixation. Any option with early weight bearing should be excluded. The Hawkin's sign appears 6-9 weeks after trauma and is indicative of vascular viability. The presence of the subchondral radiolucent band of the talar dome is 100% sensitive, but only 57.7% specific to rule out avascular necrosis of the talus. The tenuous, retrograde blood supply of the talus is of greater concern with fractures of the talar neck.

A 54 year old gentlemen presents to your office today complaining of bilateral knee pain. He was diagnosed with osteoarthritis in both his knees after he was referred by his primary care provider to see an orthopedic surgeon. He was given a prescription for an antiinflammatory medication and told that he may need knee replacement surgery one day. He does not like taking medications and has come to see you regarding non-pharmacologic treatment options. Which of the following statements regarding exercise and osteoarthritis is true? A. Aerobic exercise should be discouraged because it will increase the patient's pain B. This patient's x-ray findings are moderate to severe therefore exercise is unlikely to be helpful C. Strengthening exercises may be helpful in preventing OA and may also alter disease progression D. Aquatic exercise has not been shown to be beneficial for patients with OA

C Answer A is incorrect: for people with OA, both high intensity and low intensity aerobic exercise have been shown to be effective in improving a patient's functional status, gait, pain and aerobic capacity. Different exercise types have different effects; thus, an individualized approach to exercise prescription is recommended, based on presenting symptoms, problems and the needs of the patient. For people with OA of the knee, land-based therapeutic exercise has been shown to reduce pain and improve physical function. Optimal exercise type or dosage has not clearly been defined. Supervised exercise classes appeared to be as beneficial as treatments provided on a one-to-one basis. Answer B is incorrect: the effectiveness of exercise is independent of the severity of x-ray findings. Answer C is correct: improvements in muscle strength and proprioception gained from exercise programs may prevent and reduce the progression of osteoarthritis. Answer D is incorrect because there is evidence to support aquatic exercise for the treatment of knee OA and/or hip, at least in the short-term. Although no long-term effects have been documented (very few studies performed at this point), the short-term benefits make it a viable option for your patients.

Which of the exercise prescription below should you advise against for an HIV infected individual with mild to moderate symptoms or CD4 count < 200? A. moderate exercise (40 -60 % VO2max) B. weight training C. intense exercise (>75% VO2 max) D. Three times per week

C Ullman reported an impaired ability to mobilize neutrophils and natural killer cells in response to 1h of exercise at 75% VO2 max. Otherwise, moderate exercise has been shown to increase CD4 counts and CD4:CD8 ratios. It has also been shown to lower anxiety and tension levels in this population by. Weight training may enhance muscle strength, bulk and function in HIV+ individuals and may mitigate muscle wasting.

Loss of lumbar lordosis, vertebral wedging, and Schmorl nodes in the lumbar spine are seen in what condition? A. spondylolysis B. spondylolisthesis C. Scheuerman disease D. all of the above

C, Scheuerman's disease is a self-limiting disease of childhood also known as idiopathic juvenile kyphosis of the spine. It describes a disease process where the verterbra grow unevenly in the sagital plane, where the posterior side grows faster than the anterior side, creating a vertical wedging between the vertebra and a kyphotic curvature to the spine. Excessive lumbar lordosis can also be seen as kyphosis progresses and Schmorl nodes or vertebral disc herniations can result as they are compressed by the vertebral wedging. Once the patient is fully grown, the bones will maintain their deformed structure and surgery is often warrented to correct the kyphosis.

Which of these is true of normal tendon structure? A. Vascular and lymphatic supply is contained in the loose connective tissue sheath of the endotenon B. Collagen type III makes up about 80% dry weight of the tendon C. Water accounts for about 70% of tendon mass D. The osseotendinous junction is the weakest part of the muscle/tendon unit E. The majority of the blood flow to the tendon midportion arises from vessels originating at the myotendinous junction

C. Vascular supply is mostly in the paratenon/synovial sheath. Collagen type I makes up 80% dry weight of the tendon Myotendinous junction is the weakest region of the muscle-tendon unit

Which of the following statements is true regarding hip flexor injury? A. In adolescents with the possible diagnosis of hip flexor pain and tenderness over the ischial tuberosity should have an x-ray to rule-out hip flexor origin avulsion B. A hop test with pain in the ipsilateral groin is indicative of a hip flexor strain C. Patients with large, palpable defects in the rectus femoris rarely need surgery D. Hip flexor strains are commonly accompanied by a tingling sensation in the anterior thigh because of irritation of the lateral femoral cutaneous nerve E. Significant weakness is usually seen on exam with most hip flexor strains

C. Isolated deformities of the rectus femoris usually cause little to no functional disability and rarely need surgical intervention. The most common site of avulsion of hip flexors is the rectus femoris at the anterior inferior iliac crest, not the ischial tuberosity which is the origin of the hamstrings (a hip extensor). A positive hop test is suspicious for a femoral neck stress fracture. Meralgia paresthetica is a condition caused by irritation of the lateral femoral cutaneous nerve (often at the inguinal ligament). Meralgia paresthetica is not commonly associated with hip flexor strains. Because the hip flexors are very strong muscles and large number of hip flexors, most strains do not cause significant weakness, but instead have pain (and perhaps subtle weakness) with resistance testing.

What is the national organization that exists to help you manage divers with decompression sickness? How do you contact them?

Call the DAN Emergency Hotline Call +1-919-684-9111 to talk to an expert in diving medicine. You may call collect. DAN medical staff is on call 24 hours a day to handle diving emergencies. When You Call the DAN Emergency Hotline: 1. Tell the operator you have a diving emergency. The operator will either connect you directly with DAN or have someone call you back at the earliest possible moment. 2. The DAN staff member may make an immediate recommendation or call you back after making arrangements with a local physician or the DAN Regional Coordinator. DAN Regional Coordinators are familiar with chamber facilities in their area, and because they're qualified in diving medicine, they make recommendations about treatment. 3. The DAN staff member or Regional Coordinator may ask you to wait by the phone while he / she makes arrangements. These plans may take 30 minutes or longer, as several phone calls may be required. This delay should not place the diver in any greater danger. However, if the situation is life-threatening, arrange to transport the diver immediately to the nearest local medical facility for immediate stabilization and assessment of his or her condition. Call the DAN Emergency Hotline (+1-919-684-9111) if you need evacuation assistance through DAN TravelAssist.

You and your friends were out fishing in the gulf and had a great day catching mackerel and grouper. You decide to have the fish for dinner and even experiment with some sushi that used the fish. By the next day, everyone in your group is experiencing nausea and vomiting and odd neurological symptoms. The cold beer you are drinking feels incredibly hot and the hot coffee feels strangely cold as you hold the cups. What is wrong and what is the treatment?

Ciguatera fish poisoning (or ciguatera) is an illness caused by eating fish that contain toxins produced by a marine microalgae called Gambierdiscus toxicus. People who have ciguatera may experience nausea, vomiting, and neurologic symptoms such as tingling fingers or toes. They also may find that cold things feel hot and hot things feel cold. Ciguatera has no cure. Symptoms usually go away in days or weeks but can last for years. People who have ciguatera can be treated for their symptoms.

A collegiate swimmer develops medial winging of the scapula. If the EMG and nerve conduction studies are abnormal, the most likely nerve roots to be involved are? 1. C7, C8, T1 2. C6, C7, C8 3. C5, C6, C7 4. C4, C5, C6 5. C3, C4, C5

Classic medial winging of the scapula is due to paralysis of the serratus anterior muscle which is supplied by the long thoracic nerve which holds the scapula to the chest wall and prevents the inferior angle of the scapula from migrating medially. It is innervated by the long thoracic nerve (C5, 6, 7). Surgical treatment often is reserved for those that don't recover by 9 months of physical therapy or up to two years afterwards, and may require muscle transfer to compensate for the loss of serratus function. Lateral winging may be caused by spinal accessory nerve palsy (CN XI, also ventral ramus C2,3,4). The nerve may be injured during neck surgery. This causes trapezius weakness, allowing the inferior pole of the scapula to migrate laterally.

A 23 year old cross country runner is complaining of a generalized papular itchy rash that occurs only during exercise. You would recommend that he: A. Use sunscreen with PABA (para-aminobenzoic acid) at least 20 minutes prior to exercise B. Use topical steroids on the rash during exercise C. Take a hot shower the night before a long run or try an antihistamine tablet one hour prior to exercise. D. Desensitize with PUVA (Psoralen + UVA)

Correct Answer, C. This athlete has cholinergic urticaria which is characterized by papular wheals with surrounding erythema occurring during and after heat exposure or exercise. A hot shower prior to a long run may deplete histamine and provide a refractory period for the athlete. H1 antihistamines are effective when taken one hour prior to activity. Sunscreen with PABA or PUVA has no effect on cholingeric urticaria

The use of a TENS (transcutaneous electric nerve stimulation) unit: A. Results in increased dorsal horn cell activity B. Most likely relieves pain via endorphin release with high frequency, low intensity modalities C. Is relatively contraindicated for a patient with an implantable cardiac defibrillator (ICD) D. Has been proven to reduce fracture pain E. Results in local analgesia that is typically long-lasting (> 1 hour) after stimulation is stopped

Correct answer is C. Cardiac pacemakers may be relatively resistant to TENS signals, but there is one published report of an ICD being triggered by use of a TENS unit. Other relative contraindications to using a TENS unit include local skin irritation and contact dermatitis. TENS results in decreased dorsal horn cell activity after stimulation. High frequency, low intensity stimulation most likely produces analgesia via the gate theory whereas high intensity, lower frequency signals work by endorphin release. Hypoanalgesia may persist for up to 5 minutes after cessation of stimulation.

Which of the following is primarily associated with endurance training? A. Ability to exert increased force B. Increased protein content of muscle fibers C. Fast to slow fiber transformation D. Reduction of muscle capillary bed E. Increased size of muscle fibers

Correct answer: (C) Fast to slow fiber transformation. Explanation: There are specific long-term adaptations to exercise training related to endurance training and strength training. The type of training affects the type of of adaptation in skeletal muscle. Endurance training results in adaptations in muscle and cardiopulmonary function that increase both maximal oxygen uptake and lactate threshold. Endurance training results in skeletal muscle mitochondrial biogenesis, fast to slow fiber transformation, expansion of the muscle capillary bed, and changes in metabolism of substrates. Resistance training is associated with increased muscle fiber size. The protein content of muscle fiber also changes in resistance training, leading to the ability to increase force. (A) This is associated with resistance training. (B) Increased protein content of muscle fibers is associated with resistance training. (D) The muscle capillary bed increases in size with endurance training. (E) Increased size of muscle fibers is the result of resistance training.

Which of the following does not cause delayed onset muscle soreness (DOMS)? A. Lactic acid accumulation in muscle tissues B. Structural damage to muscle fibers C. Eccentric exercise D. Swelling on a cellular level which may activate and sensitize afferent nerve endings around damaged muscle fibers E. Training at an intensity greater than customary

Correct: A Delayed onset muscle soreness (DOMS) describes pain 24-72 hours after unaccustomed exercise. It usually resolves in several days to a week after onset. Previously, lactic acid accumulation at the muscle site was thought to cause DOMS but it is now known that lactic acid is rapidly cleared and is not present at the time of DOMS. Studies have demonstrated that eccentric activities produce more muscle damage and more DOMS than concentric activities and this damage has been shown to involve structural damage to muscle banding patterns including disruption of sarcomere Z lines. Prostaglandin induced swelling has been demonstrated to sensitize afferent nerve fibers of muscle connective tissue which transmit sensation of dull pain to the central nervous system.

Pain originating from the facet joint complex is a common cause of back pain. The purpose of the facet joint in its protection of the lumbar intervertebral disk is best characterized as which of the following: A. protection against axial rotation and loading B. protection against shearing forces C. protection against anterior translocation D. protection against caudal translocation

Correct: A Facet joints of the cervical region of the spine are oriented primarily in the coronal plane to resist axial rotation and loading. The thoracic spine lays in an intermediate position and the lumbar spine transitions to a sagittal orientation to protect against rotatory forces. The lumbar facets can resist rotation while the coronal position of the cervical facets allows a great deal of rotation

A 13 year old female with a history of Legg-Perthes disease as a child presents to your clinic with worsening chronic left hip and groin pain. She reports occasional catching or locking. On exam she has pain with passive range-of-motion and reduced internal rotation and abduction. On MRI of the hip, you find which of the following: A. Osteochondritis Dissecans of the femoral head B. Labral Tear C. Normal hip D. Arthritis

Correct: A Legg-Calve-Perthes disease is idiopathic avascular necrosis of the femoral head in young people. It presents is at age 4 to 8 and is more common in males. It typically is self limited and undergoes resorption, collapse, followed by repair of the capital femoral epiphyses. The result is impaired development of the hip joint. Half of children who develop Legg-Perthes disease will develop osteoarthritis later in life. Additionally, some patients do not spontaneously resolve and develop an osteochondral fragment which fails to unite with the rest of the femoral head.

A baseball player was hit in the upper thigh by a line drive and he complains of swelling and pain immediately. You note a large hematoma present in the location of the injury. You begin your evaluation by palpating the borders of the femoral triangle. Which of the following structures does not form a border of the triangle? A. Medial border of the adductor brevis B. Inguinal ligament C. Medial border of the adductor longus D. Medial border of the sartorius

Correct: A Several important structures lie in the femoral triangle, including the femoral artery, vein and nerve. It is also the location of compression for tamponade of the femoral artery to the leg where the femoral artery lies over the head of the femur. While the brevis does run in parallel to the adductor longus, it lies deep to the adductor magnus and therefore does not form a border of the triangle.

Which of the following is a known result of resistance training? A. In elderly patients, improved balance, mobility, and strength to perform ADLs. B. Decrease in bone mass and in strength of connective tissue. C. Decrease in lean body mass. D. Large improvement in cardiorespiratory fitness E. Decrease in glucose tolerance and lipid profiles when resistance is a component of circuit training.

Correct: A Studies have shown that strength training in the elderly can improve balance, mobility and strenth to perform ADLs. Weight bearing exercise can increase bone mass and density as well as increasing lean body mass. Glucose tolerance and lipid profiles also improve with regular activity. There is only slight improvement in cardiorespiratory fitness when resistance training is a part of circuit training.

The anterior tibialis is the main dorsiflexor of the ankle, it originates on the anterolateral tibia and interosseus membrane and inserts on: A. Medial cuneiform and base of 1st metatarsal B. All 3 cuneiform bones, and the base of the 2nd metatarsal C. Navicular bone D. Anterior talus

Correct: A The anterior tibialis muscle is the largest muscle in the anterior leg, and the main occupant of the anterior compartment. In addition to dorsiflexing the ankle, the muscle also adducts and inverts the foot. The tendon crosses the anterior to the ankle joint just medial to the midline, then sweeps across the dorsum of the foot medially to insert on the plantar surface of the medial cuneiform and base of the 1st metatarsal. The muscle is innervated by the L4 nerve root contained in the deep peroneal nerve. Rupture of this tendon can occur and is typically seen in individuals over age 45 after a forceful plantarflexion of the foot.

Which of the following statements related to blood loss and anemia in athletes is false? A. Rupture of red blood cells occurs in the capillaries of the feet during endurance running events B. Hematuria in marathon runners lasts for several weeks after the marathon event C. Well trained athletes with Sickle Cell trait perform similarly to athletes with normal hemoglobin at aerobic and anaerobic sports D. Iron deficiency anemia is the most common form of anemia found in athletes E. A drop in the hematocrit in well trained endurance athletes can be a physiologic adaptation to exercise not requiring treatment

Correct: B "Sports anemia" has been considered an "innocent side effect of a healthy hobby." "E" describes physiologic anemia in athletes, which is the increase in both blood volume and hematocrit with endurance training. It does not require treatment. True iron deficiency anemia in athletes is also an important issue. Iron deficiency causing anemia requires treatment. Studies show that this is the most common reason for anemia in athletes. There are many possible causes of iron deficiency anemia in our athletic population. Poor intake of iron rich foods, menstrual losses, disordered eating in general, and a number of mechanical mechanisms of red cell destruction (capillaries in the feet and bladder motion during sports) can contribute. Low iron stores without anemia seems to affect performance as well, and iron supplementation in these athletes has become a common and successful protocol among college and elite endurance athletes, although the research evidence supporting this is running behind the observed benefits noted by team physicians and athletes. Studies show that athletes with sickle cell trait are more susceptible to exercise related collapse and should be informed of that risk. But they should not be disqualified from sports simply due to their sickle cell trait status. Trained athletes with sickle cell trait perform as well as trained athletes without sickle cell trait. Hematuria associated with endurance events should only last 48 hours or so. If hematuria persists in athletes, it should be addressed in a similar fashion as in our general adult population. Unexplained hematuria can result from a serious condition and should not be ignored.

A 16 year old male snowboarder had an accident during the Olympic competition. It was significant enough that it was decided to transport him to the hospital. En route, he complained of left shoulder pain, but remained hemodynamically stable during transport. At the hospital, his hemoglobin remained normal and stable throughout. CT scanning with contrast revealed a Grade II Splenic injury. Things to consider during his initial evaluation, management and disposition include: A. The spleen is rarely injured during sport B. Non-operative management would be preferred C. Splenic rupture is of minor concern in this patient D. Ultrasound is the preferred method of imaging in stable patients E. He should be vaccinated immediately

Correct: B Answer A is not correct. The spleen is the most commonly injured abdominal organ in sports. Answer B is the best answer for several reasons. The predominant reasons include the fact that he is an adolescent who is hemodynamically stable. Preservation of the spleen is always preferable in the long term. And since his competitive season is likely over for a while after the Olympics, he would be an excellent candidate for non-operative management which is the currently preferred method of management. Healing can take several months. Answer C is incorrect because delayed splenic rupture is the greatest concern after 48 hours in the non-operatively managed patient. Answer D is incorrect because CT scanning has been shown to be superior to US. If the patient is unstable, portable ultrasound would then be the preferable imaging method, but is not pertinent in this scenario. Answer E is correct if the patient is thought to need an urgent splenectomy. There was no indication that emergent surgery was needed, and therefore is not the best answer.

A day after being struck with a pitched ball on the ulnar aspect of the left wrist and hand, a professional baseball player develops "pins and needles" in the small and ulnar half of his ring fingers. He finds it extremely difficult to grab the bat to participate in batting practice. After x-rays demonstrate no acute abnormalities of the left wrist and hand, he is diagnosed with Guyon's canal syndrome. What two bones form Guyon's canal? A. Pisiform and Triquetrum B. Pisiform and Hamate C. Hamate and Lunate D. Triquetrum and Lunate

Correct: B Answer: B Guyon's canal syndrome is entrapment of the ulnar nerve as it passes through a tunnel in the wrist called Guyon's canal. The canal is formed by the most lateral bones of the proximal and distal carpal rows, the pisiform and hamate respectively, and the ligament that connects them. The ulnar nerve is accompanied by the ulnar artery as it passes through this canal. Symptoms can include a sensation of pins and needles in the small and ulnar half of the ring fingers, decreased sensation in the same distribution as well as weakness of the small muscles of the palm and the muscle that pulls the thumb towards the palm.

A 22 year old male wrestler presents to your clinic after falling awkwardly in a match approximately 4 hours earlier in the day injuring his left wrist. The patient appears uncomfortable and states the pain has been getting worse since the time of the injury despite ice and immobilization. On exam, he has swelling and is tender over the distal radius. His neurovascular exam is intact but he is unwilling to allow extension of his wrist or fingers because of pain. An x-ray is done and shows a minimally displaced extraarticular fracture of the distal radius. Which of the following complications of this injury is most likely at this time? A. Stretch injury of the median nerve B. Compartment syndrome C. Malunion D. Complex regional pain syndrome

Correct: B Correct answer B: Compartment syndrome of the antebrachium may present with pain out of proportion to the injury, tenseness of the forearm, swelling, and pain with passive movement of the fingers and wrist. Early on, the patient usually has an intact radial pulse and good refill as these changes tend to occur late after significant tissue damage has already occurred. Median nerve injury may occur as pressures within the compartment continue to rise.

An otherwise healthy 16 year old male gymnast presents with a three month history of non-radiating bilateral low back pain that worsens when he does back hand springs. On physical examination, his pain worsens with extension based maneuvers and he has markedly decreased bilateral hamstring flexibility. There is no evidence of spondylolisthesis on plain x-rays. A bone scan with SPECT and a thin-slice CT confirm your diagnosis. What rehab program would you prescribe? A. Extension-biased spinal stabilization and quadriceps flexibility exercises B. Flexion-biased spinal stabilization and hamstring flexibility exercises C. Extension-biased spinal stabilization and hamstring flexibility exercises D. Plyometric exercise program E. A rehab program is not indicated for this condition

Correct: B Correct answer is B. This athlete's most likely diagnosis is lumbar spondylolysis, a stress fracture of the par interarticularis. The pars interarticularis is the region of the spinal lamina between the superior and inferior articulating processes. Spondylolysis often occurs due to repetitive hyper-extension and axial rotation stresses on the lumbar spine. This problem is very common in gymnasts. Lumbar spondylolysis most commonly occurs at the L5 level. Bilateral spondylolysis is more common than unilateral spondyloyisis and can lead to spondylolisthesis (slippage of one vertebral body on another). On physical examination the patient may have vertebral paraspinal muscle tenderness at the affected level, limited painful range of motion in both flexion and extension, and significant worsening of the low back pain with extension-based spine testing maneuvers. Up to 80% of patients with lumbar spondylolysis will have associated decreased hamstring flexibility. Initial treatment is conservative management including complete rest from the athlete's sport, a therapy program focusing on core strengthening, flexion-biased spinal stabilization and hamstring flexibility and possibly lumbar spine bracing. An extension based therapy program may exacerbate the athlete's symptoms.

A 17 year old football player tackles an opposing player and sustains a flexion injury of his neck. He falls to the ground. The ambulance is summoned and he is boarded and taken to the hospital. He is found to have an injury to the anterior spinal cord of his neck. Which of the following clinical findings match this lesion? A. Loss of motor function and position sense on the same side of the body as the lesion and loss of pain and sensation on the opposite side of the body as the lesion B. Bilateral lower extremity paralysis that is greater than the upper extremity paralysis. Bilateral loss of pain and temperature sensation, vibratory and proprioception is intact C. Weakness in both upper extremities that is more severe than the weakness in both lower extremities. Sacral function is spared D. After the period of spinal shock has resolved, the patient has no motor or sensory activity below the level of the lesion

Correct: B Selection A is incorrect as it describes a Brown-Séquard lesion. Only one side of the cord is affected and there is loss of motor function and position sense on the same side with pain and sensation on the opposite side. This is a fairly rare lesion but has the best prognosis as far as patient recovery. Selection B is correct. Anterior cord lesion often happens after a flexion injury and unfortunately has a poor prognosis. Lower extremities are usually affected with paralysis greater than the upper extremities. Temperature sensation, vibratory sensation and proprioception are intact. Selection C describes a central cord lesion. This type of lesion most often happens with the hyperextension, not a hyperflexion injury, and can happen in elderly people with spondylosis who falls. Selection D is also incorrect and describes complete severing of the spinal cord. There is loss of both motor and sensory function below the level of the lesion. The bulbocavernosus reflex must be present to confirm that the spinal shock period is over.

Which of the following statements regarding sickle cell trait athletes is true? A. Sickle cell trait, in contrast to sickle cell disease, has little to no mortality in athletes B. Any cramping, struggling, or collapse in a sickle-trait athlete must be considered sickling - a medical emergency - until proven otherwise C. The symptoms of exertional sickling and heat illness (heat stroke or heat cramping) are not distinguishable D. Acclimation to intense training, increased hydration, and increased rest afford no protection to sickling in athletes

Correct: B Sickle cell trait is common and generally benign, but in athletes it can be fatal if appropriate steps are not taken to prevent and treat its sequelae. There have been several deaths caused by exertional sickling in college football players and numerous more in military recruits. During strenuous activity, sickling is promoted by displacement of oxygen from hemoglobin S by lactic acidosis and high tissue temperatures, by increased concentration of hemoglobin S through dehydration, and by decrease in blood oxygen due to muscle demand. These sickled red blood cells get jammed in blood vessels and cause collapse from ischemic rhabdomyolysis. Cardiac arrhythmias and acute renal failure, known complications of rhabdomyolysis, can cause death. The symptoms and signs of sickling are unique and can easily be distinguished from those of heat illness. Heat cramping may have early warning signs such as twitching or twinges in tired muscles, whereas sickling hits suddenly without warning. Heat cramping pain is the severe pain of sustained, full contraction of muscles, whereas sickling pain is a milder, ischemic pain from muscles working on diminished blood supply. Heat cramping athletes "hobble to a halt" as their fully contracted muscles no long work, whereas sickling athletes "slump to a stop" as their legs become weak and can no longer hold them up. The heat cramping athlete is often yelling in pain as their muscles are rock hard in full contraction, whereas the sickling athlete lies fairly still saying their legs won't hold him up. The muscles look and feel normal. Major heat cramping can take hours to resolve even if resting in a cool place and being treated with stretching, massage, and IV fluids. A sickling athlete feels normal after about 15 minutes of sitting in a cold tub, drinking fluids, and receiving supplemental oxygen. Sicking risk is increased by anything that increases the difficulty of the exercise, for example, hot weather, dehydration, high altitude, or asthma. Strategies that emphasize acclimation to conditioning and lifting regimens, modifying drills to allow for adequate rest between repeated sprints, good hydration, and athlete recognition of mild signs can decrease sickling in sickle c

Which pair of objective findings is most suggestive of increased intracranial pressure? A. Tachycardia, low blood pressure B. Bradycardia, elevated blood pressure C. Tachycardia, elevated blood pressure D. Bradycardia, low blood pressure

Correct: B Signs and symptoms that suggest a rise in ICP including headache, nausea, vomiting, ocular palsies, altered level of consciousness, and papilledema. If mass effect is present with resulting displacement of brain tissue, additional signs may include pupillary dilatation, abducens (CrN VI) palsies, and the Cushing"s triad. The Cushing"s triad involves an increased systolic blood pressure, a widened pulse pressure, bradycardia, and an abnormal respiratory pattern.

A 17 year old male high school baseball pitcher presents to your sports medicine clinic for review of a MRI ordered by another physician. The pitcher has pain in his throwing shoulder. The MRI demonstrates bone marrow edema and cortical flattening suggestive of a Hill-Sachs lesion in the proximal humerus with subchondral sclerosis in the posterosuperior aspect of the glenoid. You would anticipate which of the following physical exam findings based on the imaging study? A. Visible atrophy of the supraspinatus and infraspinatus with muscular weakness on testing B. Posterior shoulder pain with passive abduction and external rotation of the affected shoulder C. Marked weakness of shoulder internal rotators D. Enlarged cervical and peri-clavicular lymph nodes

Correct: B The MRI findings of bone marrow edema and cortical flattening suggestive of a Hill-Sachs lesion in the proximal humerus with subchondral sclerosis in the posterosuperior aspect of the glenoid may be seen in patients with internal impingement of the shoulder. Internal impingement may be clinically assessed with the "relocation test of Jobe." In patients with internal impingement abducting and externally rotating the affected shoulder produces posterior shoulder pain. Posteriorly directed humeral head pressure, "relocation", with the patient's shoulder in abduction and external rotation relieves the pain. Atrophy of supraspinatus and infraspinatus would not be expected findings. Internal impingement is typically seen in overhead throwers who generally have increased strength of shoulder internal rotators. Lymph node enlargement is not seen with this condition.

Which statement is true regarding exercise-induced anaphylaxis? A. Pre-treatment with anti-histamines is effective to reduce the occurrence rate B. Pre-treatment with NSAID's or aspirin is effective to reduce the occurrence rate C. A common trigger is running within a couple of hours after ingesting a meal D. Initial treatment is immediate administration of anti-histamines and steroids E. Re-occurrence is rare so affected athletes can run alone with little risk

Correct: C Affected athletes should never run alone as there are no proven measures to prevent an attack. NSAID's/aspirin are common triggers for such an attack.and their use should be avoided before exercising. The initial step in management is always epinephrine - preferably IM.

You are participating as a volunteer physician in a preparticipation physical examination night for incoming freshman athletes at a local college. The examinations are station-based and quite busy with 4 physicians to evaluate 80 athletes. You see a female cross country runner with a height of 5'6"; weight 100 lbs; pulse of 45 and respirations of 18. She says she has always been thin and her mother is also thin. Her last menstrual period was more than a year ago. She has been told that the lack of menses is of no concern since she is an athlete. Physical exam is unremarkable except for the very thin stature. Your recommendations are: A. Cleared for participation, but needs nutrition evaluation B. Not cleared for participation until nutrition evaluation C. Not cleared for participation until further evaluation by team physician and completion of nutrition evaluation D. Order labs to include estradiol, LH, FSH, TSH and if normal, clear her for participation

Correct: C Amenorrhea and extreme low body weight are hallmarks for eating disorder. This athlete needs more evaluation than can be performed at a typical pre-participation examination. Further workup including a more detailed medical and menstrual history by the team physician and input from other experts such as a nutritionist would be needed before this athlete could be safely cleared for participation (correct answer C).

You are covering a weightlifting tournament and the competitor "misses the snatch" (the olympic lift in which the athlete attempts to move a loaded barbell from the floor to an overhead postion in one fluid motion). You notice through your direct observation of the lift (and subsequent review of the video tape) that the loaded barbell (which weighed around 250 pounds) came down on the athlete's neck. The athlete was able to walk off the competition platform under their own power before you could reach the individual. Off the platform, the athlete complains of a sore neck and "nothing else." He denies radicular symptoms, limb weakness, headache, or parathesias. Your exam reveals normal peripheral neurological exam (DTR, sensation, strength) but some paracervical muscle sorenessand spinous process tenderness. Which of the following is most likely diagnosis? A. C3-C4 cervical subluxation B. Rupture of the Ligamentum Flava C. Clay Shoveler's fracture D. Paraspinal muscle strain E. Thoracic Outlet Syndrome

Correct: C Due to the ballistic nature of the lift and the weight of the loaded bar, one must consider all the above as possible diagnoses but must rule out the significant injuries first. The most serious are the subluxation and fracture. Thoracic outlet syndrome is a more chronic condition while the ligamnetum flava is virtually impossible to rupture. Cervical subluxation would present with limited ROM and significant neurlogical findings, while Clay shoveler's fracture would not. A simple radiograph would also confirm if either was present.

A 22 year old male American football player suffers a hyperpronation injury of the right forearm and this results in a first-time dorsal-ulnar dislocation of the distal radioulnar joint (DRUJ). Fracture is ruled out by radiographs and adequate closed reduction is achieved. How should this injury be managed? A. Thumb spica splint for 2 weeks B. Short arm cast for 4 weeks C. Long arm cast for 6 weeks D. Orthopedic referral for arthrodesis

Correct: C The long arm cast for 6 weeks is the correct management for a distal radioulnar joint dislocation without fracture. Both the thumb spica a) and short arm b), would not provide the correct immobilization of supination and pronation of the forearm that is necessary. Orthopedic referral d), is also incorrect as the question indicates this is not a recurrent injury and adequate reduction is achieved without fracture

A 29 year old hockey player complains of one day of eye pain. The pain began suddenly while he was sharpening his skates without using eye protection. Upon exam, his visual acuity is intact, as are extraocular movements. Seidel's test is negative. Inspection shows scleral erythema and a ½ mm brownish staining at 12 o'clock superior to the iris. In addition to standard management and removal of this corneal foreign body, what steps must be taken in this case? A. Expansion of microbial coverage to include atypical organisms B. Patching for 1 week to decrease spreading inflammation C. Removal of products of iron oxidation from the cornea D. Repair of corneal perforation in the operating room E. Prolonged course of mydriatics

Correct: C The practitioner in this case has discovered a rust ring. This is the product of oxidation of iron materials that are embedded traumatically in the cornea. Not only must the iron shard be removed, but the rust ring must be removed to prevent vascularization degeneration of the cornea. There is no need presently to treat with expanded spectrum antibiotics. Patching is not necessary and may do harm. Seidel's test is negative, which rules out corneal perforation. Prolonged mydriatics have no further role in management of rust rings.

A 16 year old male football player presents to your office with acute onset of mid-thoracic back pain which began immediately after being struck in the back during a football game the previous evening. On exam, you note an area of point tenderness immediately lateral to the midline in the mid-thoracic region of the athlete's back. Other than some moderate paravertebral muscle spasm, he has no other physical findings. Radiographic evaluation reveals a nondisplaced transverse process fracture. Which of the following are appropriate management options for this athlete? A. Immediate immobilization on a back board and transfer to the hospital for neurosurgical evaluation B. Referral for fitting of a clam-shell type back brace C. Use of local ice, analgesics and anti-inflammatory medication, with return to activity as tolerated D. MRI evaluation to assess spinal cord compromise E. Disqualification from participation in collision sports for a minimum of six months

Correct: C Transverse process fractures typically occur in sports as a result of a collision, usually involving rotation or extension. Athletes can typically relate immediate onset of sharp pain associated with the collision. Because of the relationship between the transverse process and other nearby structures such as ribs and paravertebral muscles, transverse process fractures are considered stable processes. As such, they require no further surgical intervention. Bracing is contraindicated in the management of these fractures, as it often adds to the patient's discomfort. The diagnosis of transverse process fracture is made through plain radiograph or CT, and additional imaging is not necessary. Because of the stable nature of the fracture, treatment is designed to decrease discomfort, and athletes can return to play when they are comfortable, often using a flak jacket for additional protection.

A 16 year old male long jumper lands awkwardly with his R knee hyperextended, collapsing in the pit. He experiences acute swelling of the R knee immediately. PMH of resolved bilateral "jumper's knee" and prominent tibial tubercles diagnosed two years ago. There is anterior deformity and swelling immediately distal to the patella. Which of the following statements is true regarding tibial tubercle apophyseal fracture? A. patients with type 2 and 3 fractures of the tibial tubercle are able to actively extend the knee against gravity several degrees B. Negative Lachman testing immediately after injury eliminates rupture of the anterior cruciate ligament as a possibility C. Fracture at the tibial apophysis can be comminuted, displaced or involve the tibial articular surface D. Osgood Schlatter's disease is not associated with tibial tubercle fracture

Correct: C Type 2 fractures involve the inferior pole of the patella and Type 3 fractures include the anterior tibial epiphysis. They may also be Salter 3 or 5 fractures depending on extent of damage to the tibial articular surface and growth plate respectively. Type 1 injuries involve the apophysis alone. The remaining patellar tendon / apophyseal unit allows terminal extension against gravity. In types 2 and 3 active terminal extension is no longer possible due to complete disruption of tubercle / patellar anchor. This mechanism of injury is compatible with severe injuries including ACL rupture, collateral ligament and meniscal injury. Tibial apophyseal fractures and ACL rupture both may exhibit acute massive hemarthrosis. Complete assessment of suspected pediatric tibial tubercle avulsion requires MR imaging. Osgood Schlatter's disease may predispose to tibial tubercle fracture. Rosenberg demonstrated with imaging that OSD is caused by tendinopathy at the anterior ossification center of the tibial tubercle and not apophysitis. One theory notes an association of OSD to subsequent comminuted fractures.

Which of the following cervical spine injuries are both considered stable non-emergent fractures? A. Flexion teardrop fracture and clay shoveler fracture B. Hangman fracture and posterior neural arch fracture C. Simple wedge fracture and flexion teardrop fracture D. Posterior neural arch fracture and simple wedge fracture

Correct: D A flexion teardrop fracture occurs when flexion of the spine, along with vertical axial compression, causes a fracture of the anteroinferior aspect of the vertebral body. For this fragment to be produced significant posterior ligamentous disruption must occur. Since the fragment displaces anteriorly, a significant degree of anterior ligamentous disruption exists. This injury involves disruption of all 3 columns, making this an extremely unstable fracture that frequently is associated with spinal cord injury. A clay shoveler fracture occurs with abrupt flexion of the neck combined with heavy upper body and lower neck muscular contraction. The fracture may also occur with direct blows to the spinous process. The fracture is located at the base of the spinous process. This fracture is considered stable since the injury involves only the spinous process and is not associated with neurological impairment. A Hangman fracture results from a hyperextension injury that fractures both of the pedicles of C2. It is considered an unstable fracture, however it seldom is associated with spinal injury, since the anteroposterior diameter of the spinal canal is greatest at this level, and the fractured pedicles allow decompression. The posterior neural arch fracture occurs when the head is hyperextended and the posterior neural arch of C1 is compressed between the occiput and the strong, prominent spinous process of C2, causing the weak posterior arch of C1 to fracture. The transverse ligament and the anterior arch of C1 are not involved, making this fracture stable. A simple wedge fracture occurs with a pure flexion injury. The nuchal ligament remains intact. There is no posterior disruption making this a stable fracture. Answer D is the only pairing of two stable fractures in the choices above.

The most effective treatment for a symptomatic dorsal carpal ganglia is A. Nothing as most ganglia resolve spontaneously and do not require treatment B. Aspiration with corticosteroid injection C. Aspiration without corticosteroid injection D. Surgery

Correct: D Ganglion cysts account for approximately 60 percent of soft tissue, tumor-like swelling affecting the hand and wrist. They usually develop spontaneously in adults 20 to 50 years of age. There is a female-to-male preponderance of 3:1. The dorsal wrist ganglion arises from the scapholunate joint and constitutes about 65 percent of ganglia of the wrist and hand. The volar wrist ganglion arises from the distal aspect of the radius and accounts for about 20 to 25 percent of ganglia. Flexor tendon sheath ganglia make up the remaining 10 to 15 percent. The cystic structures are found near or are attached to tendon sheaths and joint capsules. The cyst is filled with soft, gelatinous, sticky, and mucoid fluid. INDICATIONS AND DIAGNOSIS Cysts are self evident, being soft and ballotable, and occur along the dorsal and volar aspects of the wrist. Most ganglia resolve spontaneously and do not require treatment. If the patient has symptoms, including pain or paresthesias, or is disturbed by the appearance, aspiration with or without injection of a corticosteroid is effective (no recurrence of the cyst) in 27 to 67 percent of patients. However, a recent randomized controlled trial between surgery and aspiration combined with methylprednisolone acetate injection plus wrist immobilization in the treatment of dorsal carpal ganglion showed the success by excision was 81.8% and by aspiration combined with methylprednisolone acetate injection plus wrist immobilization was 38.46%. The p-value was 0.047 by Fisher exact test. The present study has clearly shown that surgical excision gave a better success rate in the treatment of dorsal carpal ganglion.

A 12 year old girl with no previous hip problems suffers an injury immediately after landing in the sand pit following setting her personal record in the long jump. She now has pain and tenderness deep within the hip over the proximal and medial femur. There is pain with passive internal and external rotation of the involved hip and with active hip flexion. The examination of the other hip is normal. The most accurate diagnosis is: A. Avulsion of the apophysis over the ischial tuberosity B. Avulsion of the apophysis over the anterior superior ischial spine C. Avulsion of the apophysis over the anterior inferior ischial spine D. Avulsion of the apophysis over the lesser trochanter E. Avulsion of the apophysis over the greater trochanter

Correct: D The most common sites of avulsion fractures of an apophysis near the hip in a skeletally immature athlete are: the anterior superior iliac spine (sartorius), the ischium (hamstrings), the lesser trochanter (iliopsoas), the anterior inferior iliac spine (abdominal rectus), and iliac crest (abdominal muscles). These are injuries resulting from sudden mechanical force and from weakness at the secondary growth site. This type of maximal effort causing an injury in an adult would most often lead to a simple muscle strain, not a fracture of a mature bone. These injuries occur in running and jumping sports during a maximal effort. Avulsion of the apophysis over the greater trochanter is rare. Presentation of apophyseal fractures is acute. Common bursitis reactions in the same areas present gradually, often with an overuse history. Plain radiographs can demonstrate these apophyseal avulsion fractures.

A high school football player presents to your clinic with his parents. They seek information about nutrition and supplements for athletes. Which of the following statements is true regarding nutrition and high intensity exercise? A. Fat is broken down to glycogen during exercise B. In regards to training in a hot, humid environment, thirst is a sensitive and reliable indicator of dehydration and estimating fluid loss C. Due to the increased demand on an athlete's body, protein supplements are necessary in addition to a healthy diet D. An athlete's diet should consist of about 60 % carbohydrates

D During exercise, fats are broken down to fatty acids which are carried to muscles and converted to ATP. b) Because the thirst mechanism lags behind the body"s need for fluid replacement, thirst is not a good initial indicator of dehydration. Fluids should be ingested before, during, and after exercise. c) Although athletes need more protein than nonathletes, athletes consume more calories and thus consume more dietary protein, fulfilling the daily protein requirements. Supplements are not needed.

When evaluating anterior knee pain, the defining characteristics of patellar tendinitis include which one of the following? A. There are findings on imaging that are "pathognomic" for patellar tendinitis B. Surgery is more effective than rehabilitation C. Patellar tendinitis is common and rarely requires treatment D. Training errors are the most common cause

D Other than age range (teens to 40s) the most common identifiable risk factors are training errors, usually tight hamstrings and quadriceps. This is a clinical diagnosis, however characteristics when imaged suggesting Patello-femoral tendinitis include osteopenia at the distal pole of the patella; tractional osteophyte in proximal patellar tendon. Ultrasound, bone scan and MR imaging identify change in the posterior proximal 3rd of the tendon. Imaging is primarily useful to rule out more significant pathology within the knee or when considering surgery Surgery no better than conservative therapy. Surgical debridement of full-thickness abnormal tissue, then rehab to eccentric training compared with rehab to eccentric training alone showed no change in Jump height, leg press strength, pain scores, return to sports with or without pain, . Common complications range from inability to return to sport at 6 and 12 months to rare tendon rupture. Treatment includes relative rest and rehabilitation

A female cross-country runner presents early in the season complaining of heel pain. She states the pain has been present for two weeks. Initially the pain only occurred with long runs but now hurts most of the time. On exam, pain is elicited by squeezing the heel. X-rays are initially unremarkable. Repeat x-rays obtained two weeks later however confirm the diagnosis. Which statement about this condition is true? A. Surgical intervention is required B. Patient should be counseled that healing is expected to take 10-12 weeks C. Patient is a increased risk of plantar fascia rupture D. Patient can expect to return to activity in 4-6 weeks E. Patient's body habitus is not a factor in this diagnosis

D. Explanation: Calcaneal stress fractures are not considered a high risk injury. They typically heal 4-6 week after injury with activity modification including crutches with weightbearing as tolerated. Surgery is usually not required and most improve prior to seasons end. Patient has a positive squeeze test suggesting bony rather than soft tissue pathology. Low weights, as often seen in cross country runners can increase the risk of stress fractures.

A 36 year old female recreational soccer player presents with insidious onset of left posterior heel pain and a limp. She is wearing flip flops because shoes make the pain worse. Examination reveals swelling and erythema of the posterior heel. There is no palpable defect in the Achilles tendon and a Thompson test is negative. The most likely diagnosis is: A. Stress fracture of the calcaneus B. Plantar fasciitis C. Achilles tendon avulsion D. Sural neuritis E. Retrocalcaneal bursitis

E Retrocalcaneal bursitis (also called Haglund's syndrome) is associated with overuse and presents with pain behind the calcaneus. Examination reveals swelling and erythema of the posterior heel. A prominence, called a "pump bump" may be noticeable. Retrocalcaneal bursitis is associated with pain and tenderness anterior to the Achilles tendon, along the medial and lateral aspects of the posterior calcaneus. Plantar flexion of the foot and/or squeezing the bursa from side to side reproduces the patient's complaint.A stress fracture of the calcaneus produces mid-calcaneal bony tenderness and occurs with acute overuse. The symptoms of plantar fasciitis include tenderness and pain underneath (plantar surface), rather than behind the heel. A pop is generally heard and felt along with a palpable defect in the tendon and a positive Thompson test with an Achilles tendon avulsion injury. Sural neuritis is rare and the result of direct trauma. A positive percussion sign over the nerve lateral to the Achilles tendon is diagnostic of sural neuritis.

You see a patient in the emergency room with an acute lateral patellar dislocation. Which of the following factors is associated with the highest risk of persistent patellar instability? 1. Younger age 2. Increased Q-angle 3. Male gender 4. Previous patellar instability event 5. Amount of lateral patellar tilt

Females (not males) have a higher incidence of patellofemoral instability due to their increased Q-angle. The Q-angle or quadriceps angle is the angle formed by the intersection of a line from the ASIS to the patella and from the patella to the tibial tubercle. Normal Q-angle in males is 14 degrees and in females is 18 degrees. A higher angle means that there is a larger lateral vector force on the patella, which predisposes to lateral patellar instability. While an increased q angle increases the chance for dislocation, a previous history of dislocation is the strongest predictor.

On the way to a recompression chamber, what is the recommended initial medical treatment?

For severe decompression illness, aggressive IV hydration is recommended. For all cases breathing 100% oxygen by non rebreather mask helps reabsorb nitrogen bubbles in the tissue.

Describe a snowboarder's ankle or snowboarder's fracture

Fracture of the lateral process of the talus caused by sudden dorsiflexion and hindfoot inversion.

Please describe the diagnosis, initial management and potential complications of a knee dislocation.

Knee dislocations are devastating injuries that must be suspected with a mechanism involving a rotational component. By definition, three of the four ligaments are disrupted (combinations of ACL, PCL, MCL, LCL). Associated injuries are common. Vascular injuries occur in 5-15% of all dislocations, but 40-50% of anterior/posterior mechanisms, due to tethering at the popliteal fossa. Nerve injuries are also common, most often the common peroneal nerve (25%) of the time. Tibial nerve injuries are less frequent. Fractures are present 60% of the time, involving the tibia or femur, most commonly. Complications are very common. It is unusual for the knee to recover to pre-injury state. Diagnosis can be tricky, as the knee may reduce spontaneously (reported as 50%). Signs of trauma with swelling, etc., may be the only first cue. If there is obvious deformity, reduce immediately, particularly if distal pulses are nonpalpable. If a dimple sign is seen (medial femoral condyle buttonhole through capsule medially), this is indicative of an irreducible posterolateral dislocation, so do not reduce closed. Physical exam of the ligaments is crucial, as appearance can be normal. Vascular exam is crucial PRE and POST reduction. Serial evaluations of pulses in dorsalis pedis and posterior tibialis is mandatory while under observation and perioperatively. This is a true orthopedic emergency. If reduction is needed on site, check pulses and splint in slight flexion at 20-30 degrees. Consider CT angiography and/or Doppler vascular studies upon arrival to ED. Vascular surgical consult should be considered. Vascular surgery takes priority if vascular compromise is present. If not compromised, monitoring pulses until considered stable, with knee immobilized, is appropriate, until reconstruction planning and surgery can take place. Complications are common and include stiffness/arthrofibrosis (35-40%), laxity/instability (35-40%), peroneal nerve injury (25%), and vascular compromise.

Patient presents with midfoot pain after playing soccer. States his foot was planted and plantar flexed when stepped on from behind. Ankle exam does not reveal laxity, but he has a positive pronation-abduction stress test. What is the likely diagnosis and how should you manage this injury?

Lisfranc injury * A condition characterized by disruption between the articulation of the medial cuneiform and base of the second metatarsal * unifying factor is disruption of the TMT joint complex * injuries can range from mild sprains to severe dislocations * may take form of purely ligamentous injuries or fracture-dislocations * ligamentous vs. bony injury pattern has treatment implications Stable injuries: cast immobilization for 8 weeks * * indications * no displacement on weight-bearing and stress radiographs and no evidence of bony injury on CT (usually dorsal sprains) * certain nonoperative candidates * nonambulatory patients * presence of serious vascular disease * severe peripheral neuropathy * instability in only the transverse plane open reduction and rigid internal fixation * indications * any evidence of instability (> 2mm shift) * favored in bony fracture dislocations as opposed to purely ligamentous injuries * outcomes primary arthrodesis of the first, second and third tarsometatarsal joints

You have a patient who presents with a boxer's fracture. What are indications for referral for ORIF, and what is acceptable for alignment?

Metacarpal neck fractures most commonly involve the fifth digit and are referred to as boxer's fractures. These fractures usually result from punching a hard object, such as a wall or another person. The fracture occurs just below the metacarpal head and the metacarpal head is displaced in a volar direction. The distal fifth metacarpal takes the brunt of the impact and breaks through the narrowest area near the neck, typically resulting in apex dorsal angulation or displacement at the fracture site. Casting or splinting is helpful to avoid further instability and to promote healing with stability at the fracture site. Apex dorsal angulation up to 40 degrees is acceptable for nonoperative management. Fractures in this location often heal with residual apex dorsal angulation, which typically is not problematic. Fractures that are markedly comminuted or angulated occasionally may require open reduction and internal fixation.

What is the most common congenital coronary anomaly in sudden cardiac death?

Origin of the left coronary artery from the right sinus of valsalva

Give the origin and insertion, action and innervation of the serratus anterior muscle

Originates on ribs 1-8. Inserts on the anteromedial border of the scapula. It retracts the scapula and holds it to the posterior thorax. It is innervated by the long thoracic nerve. Muscle dysfunction or paralysis leads to medial winging of the scapula.

Posterior shoulder tightness can lead to a glenohumeral internal rotation deficit (GIRD). This has been linked most closely to which of the following shoulder pathologies? 1. Internal impingement2. Humeral avulsion of the glenohumeral ligament3. Subacromial impingement4. Bicep tendinitis5. Hill-Sachs lesion

Repetitive overhead throwing can lead to posterior capsular stiffness and relative loss of internal rotation (GIRD). This may shift the contact point posterior and superior on the glenoid, leading to internal impingement where the greater tuberosity impinges on the posterosuperior labrum and posterior rotator cuff when the arm is abducted and externally rotated. Initial treatment involves posterior capsular stretching.

Your 24-year-old stud baseball pitcher complains of vague right shoulder pain. It is most noticeable at the late cocking phase of windup. On physical exam the patient is noted to have weakness with external rotation. EMG findings are consistent with quadrilateral space syndrome. Along with the deltoid, what other muscle is affected?1. Teres major2. Teres minor3. Pectoralis major4. Supraspinatus5. Subscapularis

The axillary nerve passes through the quadrilateral space on its path to innervate the teres minor and deltoid and provide sensation to the lateral arm. This syndrome is caused by compression of the posterior humeral circumflex artery and axillary nerve or one of its major branches in the quadrilateral space. Forward flexion and/or abduction and external rotation of the humerus aggravate the symptoms. In some individuals, especially, throwers, the nerve can become irritated from the repetitive motion leading to nerve dysfunction. The condition is usually self-limited.

Which of the following statements is true regarding skin infection in athletes? A. Rifampin is the first line treatment for MRSA infections B. Any skin wound that is suspicious for Staphylococcus infection should be cultured C. The gold standard treatment of MRSA is appropriate oral antibiotics D. First line treatment of MRSA should be topical antibiotics E. Special cleaning of locker room, equipment and playing area is needed if MRSA is diagnosed in F.

The best answer is B. Any wound that is suspicious for Staphylococcus infection should be cultured. Suspicious features include: chief complaint of "spider bite" or a non-healing wound and "pus under pressure" on examination. Generally CA-MRSA wounds are clinically indistinguishable from methicillin-sensitive S. aureus and streptococcal skin infections. Rifampin may be used in combination with other antibiotics for a synergistic effect, but should never be used alone. The gold standard treatment of CA-MRSA infections is incision and drainage. The transmission of MRSA is primarily skin-to-skin; not via fomites, thus good hygiene is most critical.

Which of the following statements is true regarding metacarpal fractures? A. Most metacarpal fractures in athletes will eventually need a surgical procedure in order to regain full function B. 5th metacarpal fractures are called "Boxer" fractures because of their common occurrence in boxers C. Splints and casts for metacarpal fractures should immobilize the proximal interphalangeal (PIP), metacarpophalangeal (MCP) and the wrist joint D. Although up to 30° of angulation is acceptable for a 5th metacarpal fracture and may be treated non-operatively, fractures with malrotation should be referred for surgical reduction E. Fractures of the hand should be treated with prolonged immobilization since early motion leads to significant risk of non-union and poor functional outcome

The best answer is D. Most metacarpal fractures can be treated non-operatively exceptions include some intra-articular fractures, open fractures, unstable fractures (which usually include transverse and short oblique fractures), severely displaced fractures and fractures with rotational deformities. "Boxer" fractures almost never occur while boxing instead they typically occur from striking a solid object with a closed fist (a more typical fracture suffered while boxing is an index finger metacarpal fracture). When immobilizing the hand, the PIP joints should be allowed motion, while the MCP should be immobilized in 70-90° of flexion. Only small amounts of fracture angulation (<10°) are acceptable for the relatively immobile 2nd and 3rd metacarpals; however the more mobile 4th and 5th metacarpals can tolerate more angulation deformity (20° and 30° respectively) before surgical reduction is needed. Malrotation is an indication for surgical treatment. Early motion is essential to good outcome for hand fractures. Delaying motion beyond 3-4 weeks increases the risk of arthrofibrosis and poor functional outcome.

Please give 2 environmental and 2 individual risk factors for decompression sickness. Think of situations or characteristics of the dive for the enviromment and medical or personal issues or behaviors for the individual.

The following environmental factors have been shown to increase the risk of DCS: the magnitude of the pressure reduction ratio - a large pressure reduction ratio is more likely to cause DCS than a small one.[20][25][31] repetitive exposures - repetitive dives within a short period of time (a few hours) increase the risk of developing DCS. Repetitive ascents to altitudes above 5,500 metres (18,000 ft) within similar short periods increase the risk of developing altitude DCS.[20][31] the rate of ascent - the faster the ascent the greater the risk of developing DCS. The US Navy Dive Manual indicates that ascent rates greater than about 20 m/min (66 ft/min) when diving increase the chance of DCS, while recreational dive tables such as the Bühlmann tables require an ascent rate of 10 m/min (33 ft/min) with the last 6 m (20 ft) taking at least one minute.[32] An individual exposed to a rapid decompression (high rate of ascent) above 5,500 metres (18,000 ft) has a greater risk of altitude DCS than being exposed to the same altitude but at a lower rate of ascent.[20][31] the duration of exposure - the longer the duration of the dive, the greater is the risk of DCS. Longer flights, especially to altitudes of 5,500 m (18,000 ft) and above, carry a greater risk of altitude DCS.[20] underwater diving before flying - divers who ascend to altitude soon after a dive increase their risk of developing DCS even if the dive itself was within the dive table safe limits. Dive tables make provisions for post-dive time at surface level before flying to allow any residual excess nitrogen to outgas. However, the pressure maintained inside even a pressurized aircraft may be as low as the pressure equivalent to an altitude of 2,400 m (7,900 ft) above sea level. Therefore, the assumption that the dive table surface interval occurs at normal atmospheric pressure is invalidated by flying during that surface interval, and an otherwise-safe dive may then exceed the dive table limits.[33][34][35] diving before travelling to altitude - DCS can occur without flying if the person moves to a high-altitude location on land immediately after diving, for example, scuba divers in Eritrea who drive from the coast to the Asmara plateau at 2,400 m (7,9

Kohler's 1. Observation 2. Open biopsy and curettage 3. Long leg non-walking cast 4. Amputation 5. Short leg walking cast

The history and radiographs are consistent with Kohler's disease, avascular necrosis of the tarsal navicular.It was thought orginially that the changes in this disease might be the result of an abnormal strain that acts on a weak navicular, but a definitive answer has not been found. Among the theories to explain the nature of this lesion, a more satisfactory one is a mechanical basis that is associated with a delayed ossification. The navicular is the last tarsal bone to ossify in children. This bone might be compressed between the already ossified talus and the cuneiforms when the child becomes heavier. Compression involves the vessels in central spongy bone, leading to ischemia, which then causes clinical symptoms. Thereafter, the perichondral ring of vessels sends the blood supply, allowing rapid revascularization and formation of new bone. The radial arrangement of the vessels of this bone is of great importance in explaining why the prognosis of this lesion is always excellent.Kohler's disease tends to affect boys more frequently than girls between ages 6-9. Treatment includes immobilization for symptom relief and observation while the navicular re-ossifies.

Give 3 physical exam cardiac auscultation maneuvers that can help distinguish hypertrophic cardiomyopathy murmurs from benign flow murmurs

Valsalva increases HCM murmur, Decreases flow murmur Standing increases HCM murmur, Decreases flow murmur Squatting Decreases HCM murmur, Increases Flow murmur

What is grip lock in gymnastics?

When the gymnast is circling the bar (giants) overlappin of leather grip against itself causes grip to lock . The hand sticks and the body continues to circle the bar, causing the forearm to fracture.


Conjuntos de estudio relacionados

Chapter 8: Therapeutic Communication

View Set

Intro to Nursing & Communication

View Set

Japanese Culture Notes, Grammar Rules, and Customs 2

View Set

Chapter 1 Essential Computer Concepts

View Set

Clinical Decision Support System

View Set

Clinical Psychology- Chapter 1: Definition & Training

View Set

Psychology Chapter 8: Cognition and Language

View Set

9th Grade Biology- Amphibians, Reptiles, and Birds

View Set

Leaseing final, Real Estate Leasing Final Review, 215 property managemanr, Intro to Prop Management Final, Real Estate Leasing, NALP

View Set

Ch 24: Management of Patients with Chronic Pulmonary Disease

View Set

Electrons, Configurations, Orbitals, Test

View Set

Premise and Conclusion Indicators

View Set

Section A: Demand Management Principles

View Set

Chapter 19 - Nursing care of the family during the postpartum period EAQS

View Set